You cannot select more than 25 topics Topics must start with a letter or number, can include dashes ('-') and can be up to 35 characters long.

445 lines
27 KiB
TeX

\documentclass[11pt]{article}
\usepackage{personal_commands}
\usepackage[italian]{babel}
\title{\textbf{Note del corso di Geometria 1}}
\author{Gabriel Antonio Videtta}
\date{27 e 31 marzo 2023}
\begin{document}
\maketitle
\begin{center}
\Large \textbf{Proprietà e teoremi principali sul prodotto scalare}
\end{center}
\begin{note}
Nel corso del documento, per $V$ si intenderà uno spazio vettoriale di dimensione
finita $n$ e per $\varphi$ un suo prodotto scalare. Analogamente si intenderà lo stesso
per $V'$ e $\varphi'$.
\end{note}
\begin{proposition} (formula delle dimensioni del prodotto scalare)
Sia $W \subseteq V$ un sottospazio di $V$. Allora vale la seguente identità:
\[ \dim W + \dim W^\perp = \dim V + \dim (W \cap V^\perp). \]
\end{proposition}
\begin{proof}
Si consideri l'applicazione lineare $f : V \to \dual W$ tale che $f(\vec v)$ è un funzionale di $\dual W$ tale che
$f(\vec v)(\vec w) = \varphi(\vec v, \vec w)$ $\forall \vec w \in W$. Si osserva che $W^\perp = \Ker f$, da cui,
per la formula delle dimensioni, $\dim V = \dim W^\perp + \rg f$. Inoltre, si osserva anche che
$f = i^\top \circ a_\varphi$, dove $i : W \to V$ è tale che $i(\vec w) = \vec w$, infatti $f(\vec v) = a_\varphi(\vec v) \circ i$ è un funzionale di $\dual W$ tale che $f(\vec v)(\vec w) = \varphi(\vec v, \vec w)$. Pertanto
$\rg f = \rg (i^\top \circ a_\varphi)$. \\
Si consideri ora l'applicazione $g = a_\varphi \circ i : W \to \dual W$. Sia ora $\basis_W$ una base di $W$ e
$\basis_V$ una base di $V$. Allora le matrice associate di $f$ e di $g$ sono le seguenti:
\begin{enumerate}[(i)]
\item $M_{\dual \basis_W}^{\basis_V}(f) = M_{\dual \basis_W}^{\basis_V}(i^\top \circ a_\varphi) =
\underbrace{M_{\dual \basis_W}^{\dual \basis_V}(i^\top)}_A \underbrace{M_{\dual \basis_V}^{\basis_V}(a_\varphi)}_B = AB$,
\item $M_{\dual \basis_V}^{\basis_W}(g) = M_{\dual \basis_V}^{\basis_W}(a_\varphi \circ i) =
\underbrace{M_{\dual \basis_V}^{\basis_V}(a_\varphi)}_B \underbrace{M_{\basis_V}^{\basis_W}(i)}_{A^\top} = BA^\top \overbrace{=}^{B^\top = B} (AB)^\top$.
\end{enumerate}
Poiché $\rg(A) = \rg(A^\top)$, si deduce che $\rg(f) = \rg(g) \implies \rg(i^\top \circ a_\varphi) = \rg(a_\varphi \circ i) = \rg(\restr{a_\varphi}{W}) = \dim W - \dim \Ker \restr{a_\varphi}{W} = \dim W - \dim (W \cap \underbrace{\Ker a_\varphi}_{V^\perp}) = \dim W - \dim (W \cap V^\perp)$. Si conclude allora, sostituendo quest'ultima
identità nell'identità ricavata a inizio dimostrazione che $\dim V = \dim W^\top + \dim W - \dim (W \cap V^\perp)$,
ossia la tesi.
\end{proof}
\begin{remark}
Si possono fare alcune osservazioni sul radicale di un solo elemento $\vec w$ e su quello del suo sottospazio
generato $W = \Span(\vec w)$: \\
\li $\vec w ^\perp = W^\perp$, \\
\li $\vec w \notin W^\perp \iff \Rad (\restr{\varphi}{W}) = W \cap W^\perp \iff \vec w \text{ non è isotropo } = \zerovecset \iff
V = W \oplus W^\perp$.
\end{remark}
\begin{definition}
Si definisce \textbf{base ortogonale} di $V$ una base $\vv 1$, ..., $\vv n$ tale per cui $\varphi(\vv i, \vv j) = 0
\impliedby i \neq j$, ossia per cui la matrice associata del prodotto scalare è diagonale.
\end{definition}
\begin{proposition} (formula di polarizzazione)
Se $\Char \KK \neq 2$, un prodotto scalare è univocamente determinato dalla sua forma quadratica $q$.
\end{proposition}
\begin{proof}
Si nota infatti che $q(\vec v + \vec w) - q(\vec v) - q(\vec w) = 2 \varphi(\vec v, \vec w)$, e quindi,
poiché $2$ è invertibile per ipotesi, che $\varphi(\vec v, \vec w) = 2\inv (q(\vec v + \vec w) - q(\vec v) - q(\vec w))$.
\end{proof}
\begin{theorem}(di Lagrange)
Ogni spazio vettoriale $V$ su $\KK$ tale per cui $\Char \KK \neq 2$ ammette una base ortogonale.
\end{theorem}
\begin{proof}
Sia dimostra il teorema per induzione su $n := \dim V$. Per $n \leq 1$, la dimostrazione è triviale. Sia
allora il teorema vero per $i \leq n$. Se $V$ ammette un vettore non isotropo $\vec w$, sia $W = \Span(\vec w)$ e si consideri la decomposizione $V = W \oplus W^\perp$. Poiché $W^\perp$ ha dimensione $n-1$, per ipotesi induttiva
ammette una base ortogonale. Inoltre, tale base è anche ortogonale a $W$, e quindi l'aggiunta di $\vec w$ a
questa base ne fa una base ortogonale di $V$. Se invece $V$ non ammette vettori non isotropi, ogni forma quadratica
è nulla, e quindi il prodotto scalare è nullo per la proposizione precedente.
\end{proof}
\begin{note}
D'ora in poi, nel corso del documento, si assumerà $\Char \KK \neq 2$.
\end{note}
\begin{theorem} (di Sylvester, caso complesso)
Sia $\KK$ un campo i cui elementi sono tutti quadrati di un
altro elemento del campo (e.g.~$\CC$). Allora esiste una base
ortogonale $\basis$ tale per cui:
\[ M_\basis(\varphi) = \Matrix{I_r & \rvline & 0 \\ \hline 0 & \rvline & 0\,}. \]
\end{theorem}
\begin{proof}
Per il teorema di Lagrange, esiste una base ortogonale $\basis'$ di $V$.
Si riordini la base in modo tale che la forma quadratica valutata nei primi elementi sia sempre diversa da zero. Allora, poiché ogni
elemento di $\KK$ è per ipotesi quadrato di un altro elemento
di $\KK$, si sostituisca $\basis'$ con una base $\basis$ tale per
cui, se $q(\vv i) = 0$, $\vv i \mapsto \vv i$, e altrimenti
$\vv i \mapsto \frac{\vv i}{\sqrt{q(\vv i)}}$. Allora $\basis'$
è una base tale per cui la matrice associata del prodotto scalare
in tale base è proprio come desiderata nella tesi, dove $r$ è
il numero di elementi tali per cui la forma quadratica valutata
in essi sia diversa da zero.
\end{proof}
\begin{remark}
Si possono effettuare alcune considerazioni sul teorema di Sylvester
complesso. \\
\li Si può immediatamente concludere che il rango è un invariante
completo per la congruenza in un campo in cui tutti gli elementi
sono quadrati, ossia che $A \cong B \iff \rg(A) = \rg(B)$, se $A$ e
$B$ sono matrici simmetriche: infatti
ogni matrice simmetrica rappresenta una prodotto scalare, ed è
pertanto congruente ad una matrice della forma desiderata
nell'enunciato del teorema di Sylvester complesso. Poiché il rango
è un invariante della congruenza, si ricava che $r$ nella forma
della matrice di Sylvester, rappresentando il rango, è anche
il rango di ogni sua matrice congruente. In particolare, se due
matrici simmetriche hanno stesso rango, allora sono congruenti
alla stessa matrice di Sylvester, e quindi, essendo la congruenza
una relazione di congruenza, sono congruenti a loro volta. \\
\li Due matrici simmetriche con stesso rango, allora, non solo
sono SD-equivalenti, ma sono anche congruenti. \\
\li Ogni base ortogonale deve quindi avere lo stesso numero
di elementi nulli.
\end{remark}
\begin{definition} (somma diretta ortogonale)
Siano i sottospazi $U$ e $W \subseteq V$ in somma diretta. Allora si dice che $U$ e $W$ sono in \textbf{somma
diretta ortogonale rispetto al prodotto scalare} $\varphi$ di $V$, ossia che $U \oplus W = U \oplus^\perp W$, se $\varphi(\vec u, \vec w) = 0$ $\forall \vec u \in U$, $\vec w \in W$.
\end{definition}
\begin{definition} (cono isotropo)
Si definisce \textbf{cono isotropo} di $V$ rispetto al prodotto scalare $\varphi$ il seguente insieme:
\[ \CI(\varphi) = \{ \v \in V \mid \varphi(\v, \v) = 0 \}, \]
\vskip 0.05in
ossia l'insieme dei vettori isotropi di $V$.
\end{definition}
\begin{note}
La notazione $\varphi > 0$ indica che $\varphi$ è definito positivo (si scrive $\varphi \geq 0$ se invece è semidefinito
positivo).
Analogamente $\varphi < 0$ indica che $\varphi$ è definito negativo (e $\varphi \leq 0$ indica che è semidefinito negativo).
\end{note}
\begin{exercise} Sia $\Char \KK \neq 2$.
Siano $\vv1$, ..., $\vv k \in V$ e sia $M = \left( \varphi(\vv i, \vv j) \right)_{i, j = 1\textrm{---}k} \in M(k, \KK)$,
dove $\varphi$ è un prodotto scalare di $V$. Sia inoltre $W = \Span(\vv 1, ..., \vv k)$. Si dimostrino
allora le seguenti affermazioni.
\begin{enumerate}[(i)]
\item Se $M$ è invertibile, allora $\vv 1$, ..., $\vv k$ sono linearmente indipendenti.
\item Siano $\vv 1$, ..., $\vv k$ linearmente indipendenti. Allora $M$ è invertibile $\iff$ $\restr{\varphi}{W}$ è non degenere $\iff$ $W \cap W^\perp = \zerovecset$.
\item Siano $\vv1$, ..., $\vv k$ a due a due ortogonali tra loro. Allora $M$ è invertibile $\iff$ nessun
vettore $\vv i$ è isotropo.
\item Siano $\vv1$, ..., $\vv k$ a due a due ortogonali tra loro e siano anche linearmente indipendenti.
Allora $M$ è invertibile $\implies$ si può estendere $\basis_W = \{\vv 1, \ldots, \vv k\}$ a una base ortogonale di $V$.
\item Sia $\KK = \RR$. Sia inoltre $\varphi > 0$. Allora $\vv 1$, ..., $\vv k$ sono linearmente
indipendenti $\iff$ $M$ è invertibile.
\item Sia $\KK = \RR$. Sia ancora $\varphi > 0$. Allora se $\vv 1$, ..., $\vv k$ sono a due a due
ortogonali e sono tutti non nulli, sono anche linearmente indipendenti.
\end{enumerate}
\end{exercise}
\begin{solution}
\begin{enumerate}[(i)]
\item Siano $a_1$, ..., $a_k \in \KK$ tali che $a_1 \vv 1 + \ldots + a_k \vv k = 0$. Vale in
particolare che $\vec 0 = \varphi(\vv i, \vec 0) = \varphi(\vv i, a_1 \vv 1 + \ldots + a_k \vv k) =
\sum_{j=1}^k a_j \varphi(\vv i, \vv j)$ $\forall 1 \leq i \leq k$. Allora $\sum_{j=1}^k a_j M^j = 0$.
Dal momento che $M$ è invertibile, $\rg(M) = k$, e quindi l'insieme delle colonne di $M$ è linearmente
indipendente, da cui si ricava che $a_j = 0$ $\forall 1 \leq j \leq k$, e quindi che $\vv 1$, ...,
$\vv k$ sono linearmente indipendenti.
\item Poiché $\vv 1$, ..., $\vv k$ sono linearmente indipendenti, tali vettori formano una base di
$W$, detta $\basis$. In particolare, allora, vale che $M = M_\basis(\restr{\varphi}{W})$. Pertanto,
se $M$ è invertibile, $\Rad(\restr{\varphi}{W}) = \Ker M = \zerovecset$, e dunque $\restr{\varphi}{W}$
è non degenere. Se invece $\restr{\varphi}{W}$ è non degenere, $\zerovecset = \Rad(\restr{\varphi}{W}) = W \cap W^\perp$. Infine, se $W \cap W^\perp = \zerovecset$, $\zerovecset = W \cap W^\perp = \Rad(\restr{\varphi}{W}) = \Ker M$, e quindi $M$ è iniettiva, e dunque invertibile.
\item Dal momento che $\vv 1$, ..., $\vv k$ sono ortogonali tra loro, $M$ è una matrice diagonale.
Pertanto $M$ è invertibile se e solo se ogni suo elemento diagonale è diverso da $0$, ossia
se $\varphi(\vv i, \vv i) \neq 0$ $\forall 1 \leq i \leq k$, e dunque se e solo se nessun vettore
$\vv i$ è isotropo.
\item Se $M$ è invertibile, da (ii) si deduce che $\Rad(\restr{\varphi}{W}) = W \cap W^\perp = \zerovecset$,
e quindi che $W$ e $W^\perp$ sono in somma diretta. Inoltre, per la formula delle dimensioni del prodotto
scalare, $\dim W + \dim W^\perp = \dim V + \underbrace{\dim (W \cap V^\perp)}_{\leq \dim (W \cap W^\perp) = 0} = \dim V$. Pertanto $V = W \oplus^\perp W^\perp$. \\
Allora, dacché $\Char \KK \neq 2$, per il teorema di Lagrange, $W^\perp$ ammette una base ortogonale $\basis_{W^\perp}$. Si conclude
dunque che $\basis = \basis_W \cup \basis_{W^\perp}$ è una base ortogonale di $V$.
\item Se $M$ è invertibile, da (i) $\vv1$, ..., $\vv k$ sono linearmente indipendenti. Siano ora
invece $\vv 1$, ..., $\vv k$ linearmente indipendenti per ipotesi. Siano $a_1$, ..., $a_k \in \KK$ tali
che $a_1 M^1 + \ldots + a_k M^k = 0$, allora $a_1 \varphi(\vv i, \vv 1) + \ldots + a_k \varphi(\vv i, \vv k) = 0$ $\forall 1 \leq i \leq k$. Pertanto, detto $\v = a_1 \vv 1 + \ldots + a_k \vv k$, si ricava che:
\[ \varphi(\v, \v) = \sum_{i=1}^k \sum_{j=1}^k a_j \, \varphi(\vv i, \vv j) = 0. \]
Tuttavia questo è possibile solo se $\v = a_1 \vv 1 + \ldots + a_k \vv k = 0$. Dal momento che
$\vv 1$, ..., $\vv k$ sono linearmente indipendenti, si conclude che $a_1 = \cdots = a_k = 0$, ossia
che le colonne di $M$ sono tutte linearmente indipendenti e quindi che $\rg(M) = k \implies$ $M$ è invertibile.
\item Poiché $\vv 1$, ..., $\vv k$ sono ortogonali a due a due tra loro, $M$ è una matrice diagonale.
Inoltre, dacché $\varphi > 0$ e $\vv i \neq \vec 0$ $\forall 1 \leq i \leq k$, gli elementi diagonali di $M$ sono sicuramente tutti diversi da zero, e quindi $\det (M) \neq 0$ $\implies$ $M$ è invertibile. Allora,
per il punto (v), $\vv 1$, ..., $\vv k$ sono linearmente indipendenti.
\end{enumerate}
\end{solution}
\begin{definition}
Data una base ortogonale $\basis$ di $V$ rispetto al prodotto
scalare $\varphi$,
si definiscono i seguenti indici:
\begin{align*}
\iota_+(\varphi) &= \max\{ \dim W \mid W \subseteq V \E \restr{\varphi}{W} > 0 \}, &\text{(}\textbf{indice di positività}\text{)} \\
\iota_-(\varphi) &= \max\{ \dim W \mid W \subseteq V \E \restr{\varphi}{W} < 0 \}, &\text{(}\textbf{indice di negatività}\text{)}\\
\iota_0(\varphi) &= \dim V^\perp &\text{(}\textbf{indice di nullità}\text{)}
\end{align*}
Quando il prodotto scalare $\varphi$ è noto dal contesto, si omette
e si scrive solo $\iota_+$, $\iota_-$ e $\iota_0$. In particolare,
la terna $\sigma(\varphi) = \sigma = (i_+, i_-, i_0)$ è detta \textbf{segnatura} del
prodotto $\varphi$.
\end{definition}
\begin{theorem} (di Sylvester, caso reale) Sia $\KK$ un campo ordinato
i cui elementi positivi sono tutti quadrati (e.g.~$\RR$). Allora
esiste una base ortogonale $\basis$ tale per cui:
\[ M_\basis(\varphi) = \Matrix{I_{\iota_+} & \rvline & 0 & \rvline & 0 \\ \hline 0 & \rvline & -I_{\iota_-} & \rvline & 0 \\ \hline 0 & \rvline & 0 & \rvline & 0\cdot I_{\iota_0} }. \]
\vskip 0.05in
Inoltre, per ogni base ortogonale, esistono esattamente
$\iota_+$ vettori della base con forma quadratica positiva,
$\iota_-$ con forma negativa e $\iota_0$ con
forma nulla.
\end{theorem}
\begin{proof}
Per il teorema di Lagrange, esiste una base ortogonale $\basis'$ di $V$.
Si riordini la base in modo tale che la forma quadratica valutata nei primi elementi sia strettamente positiva, che nei secondi elementi sia strettamente negativa e che negli ultimi sia nulla. Si sostituisca
$\basis'$ con una base $\basis$ tale per cui, se $q(\vv i) > 0$,
allora $\vv i \mapsto \frac{\vv i}{\sqrt{q(\vv i)}}$; se
$q(\vv i) < 0$, allora $\vv i \mapsto \frac{\vv i}{\sqrt{-q(\vv i)}}$;
altrimenti $\vv i \mapsto \vv i$. Si è allora trovata una base
la cui matrice associata del prodotto scalare è come desiderata nella
tesi. \\
Sia ora $a$ il numero di vettori della base con forma quadratica
positiva, $b$ il numero di vettori con forma negativa e $c$ quello
dei vettori con forma nulla. Si consideri $W_+ = \Span(\vv 1, ..., \vv a)$, $W_- = \Span(\vv{a+1}, ..., \vv b)$, $W_0 = \Span(\vv{b+1}, ..., \vv c)$. \\
Sia $M = M_\basis(\varphi)$. Si osserva che $c = n - \rg(M) = \dim \Ker(M) = \dim V^\perp = \iota_0$. Inoltre $\forall \v \in W_+$, dacché
$\basis$ è ortogonale,
$q(\v) = q(\sum_{i=1}^a \alpha_i \vv i) = \sum_{i=1}^a \alpha_i^2 q(\vv i) > 0$, e quindi $\restr{\varphi}{W_+} > 0$, da cui $\iota_+ \geq a$.
Analogamente $\iota_- \geq b$. \\
Si mostra ora che è impossibile che $\iota_+ > a$. Se così infatti
fosse, sia $W$ tale che $\dim W = \iota_+$ e che $\restr{\varphi}{W} > 0$. $\iota_+ + b + c$ sarebbe maggiore di $a + b + c = n := \dim V$. Quindi, per la formula di Grassman, $\dim(W + W_- + W_0) = \dim W +
\dim(W_- + W_0) - \dim (W \cap (W_- + W_0)) \implies \dim (W \cap (W_- + W_0)) = \dim W +
\dim(W_- + W_0) - \dim(W + W_- + W_0) > 0$, ossia esisterebbe
$\v \neq \{\vec 0\} \mid \v \in W \cap (W_- + W_0)$. Tuttavia
questo è assurdo, dacché dovrebbe valere sia $q(\v) > 0$ che
$q(\v) < 0$, \Lightning. Quindi $\iota_+ = a$, e analogamente
$\iota_- = b$.
\end{proof}
\begin{definition}
Si dice \textbf{base di Sylvester} una base di $V$ tale per cui la
matrice associata di $\varphi$ sia esattamente nella forma
vista nella dimostrazione del teorema di Sylvester. Analogamente
si definisce tale matrice come \textbf{matrice di Sylvester}.
\end{definition}
\begin{remark} \nl
\li Si può dunque definire la segnatura di una matrice simmetrica
come la segnatura di una qualsiasi sua base ortogonale, dal
momento che tale segnatura è invariante per cambiamento di base. \\
\li La segnatura è un invariante completo per la congruenza nel caso reale. Se infatti due matrici hanno la stessa segnatura, sono
entrambe congruenti alla matrice come vista nella dimostrazione
della forma reale del teorema di Sylvester, e quindi, essendo
la congruenza una relazione di equivalenza, sono congruenti
tra loro. Analogamente vale il viceversa, dal momento che ogni
base ortogonale di due matrici congruenti devono contenere gli
stessi numeri $\iota_+$, $\iota_-$ e $\iota_0$ di vettori
di base con forma quadratica positiva, negativa e nulla. \\
\li Se $\ww 1$, ..., $\ww k$ sono tutti i vettori di una base
ortogonale $\basis$ con forma quadratica nulla, si osserva che $W = \Span(\ww 1, ..., \ww k)$ altro non è che $V^\perp$ stesso. Infatti, come
visto anche nella dimostrazione del teorema di Sylvester reale, vale
che $\dim W = \dim \Ker (M_\basis(\varphi)) = \dim V^\perp$. Inoltre,
se $\w \in W$ e $\v \in V$, $\varphi(\w, \v) = \varphi(\sum_{i=1}^k
\alpha_i \ww i, \sum_{i=1}^k \beta_i \ww i + \sum_{i=k+1}^n \beta_i \vv i)
= \sum_{i=1}^k \alpha_i \beta_i q(\ww i) = 0$, e quindi
$W \subseteq V^\perp$, da cui si conclude che $W = V^\perp$. \\
\li Vale in particolare che $\rg(\varphi) = \iota_+ + \iota_-$, mentre
$\dim \Ker(\varphi) = \iota_0$, e quindi $n = \iota_+ + \iota_- + \iota_0$. \\
\li Se $V = U \oplusperp W$, allora $\iota_+(\varphi) = \iota_+(\restr{\varphi}{V}) + \iota_+(\restr{\varphi}{W})$, e
analogamente per gli altri indici.
\end{remark}
\begin{definition} (isometria)
Dati due spazi vettoriali $(V, \varphi)$ e
$(V', \varphi')$ dotati di prodotto scalare sullo stesso campo $\KK$, si dice che
$V$ e $V'$ sono \textbf{isometrici} se esiste un isomorfismo
$f$, detto \textit{isometria}, che preserva tali che prodotti, ossia tale che:
\[ \varphi(\vec v, \vec w) = \varphi'(f(\vec v), f(\vec w)). \]
\end{definition}
\begin{exercise} Sia $f : V \to V'$ un isomorfismo. Allora $f$ è un'isometria $\iff$ $\forall$ base $\basis = \{ \vv 1, \ldots, \vv n \}$ di $V$, $\basis' = \{ f(\vv 1), \ldots, f(\vv n) \}$ è una base di $V'$ e $\varphi(\vv i, \vv j) = \varphi'(f(\vv i), f(\vv j))$ $\forall 1 \leq i, j \leq n$ $\iff$ $\exists$ base $\basis = \{ \vv 1, \ldots, \vv n \}$ di $V$ tale che $\basis' = \{ f(\vv 1), \ldots, f(\vv n) \}$ è una base di $V'$ e $\varphi(\vv i, \vv j) = \varphi'(f(\vv i), f(\vv j))$ $\forall 1 \leq i, j \leq n$.
\end{exercise}
\begin{solution} Se $f$ è un'isometria, detta $\basis$ una base di $V$, $\basis' = f(\basis)$ è una base di $V'$
dal momento che $f$ è anche un isomorfismo. Inoltre, dacché $f$ è un'isometria, vale sicuramente che
$\varphi(\vv i, \vv j) = \varphi'(f(\vv i), f(\vv j))$ $\forall 1 \leq i, j \leq n$. \\
Sia ora assunto per ipotesi che $\forall$ base $\basis = \{ \vv 1, \ldots, \vv n \}$ di $V$, $\basis' = \{ f(\vv 1), \ldots, f(\vv n) \}$ è una base di $V'$ e $\varphi(\vv i, \vv j) = \varphi'(f(\vv i), f(\vv j))$ $\forall 1 \leq i, j \leq n$. Allora, analogamente a prima, detta $\basis = \{ \vv 1, \ldots, \vv n \}$ una base di $V$, $\basis' = f(\basis)$ è una base di $V'$, e in quanto tale,
per ipotesi, è tale che $\varphi(\vv i, \vv j) = \varphi'(f(\vv i), f(\vv j))$ $\forall 1 \leq i, j \leq n$. \\
Sia infine assunto per ipotesi che $\exists$ base $\basis = \{ \vv 1, \ldots, \vv n \}$ di $V$ tale che $\basis' = \{ f(\vv 1), \ldots, f(\vv n) \}$ è una base di $V'$ e $\varphi(\vv i, \vv j) = \varphi'(f(\vv i), f(\vv j))$ $\forall 1 \leq i, j \leq n$. Siano $\v$, $\w \in V$. Allora $\exists a_1$, ..., $a_n$, $b_1$, ..., $b_n \in \KK$
tali che $\v = a_1 \vv 1 + \ldots + a_n \vv n$ e $\w = b_1 \vv 1 + \ldots + b_n \vv n$. Si ricava pertanto
che:
\[ \varphi'(f(\v), f(\w)) = \sum_{i=1}^n \sum_{j=1}^n a_i b_j \, \varphi'(f(\vv i), f(\vv j)) =
\sum_{i=1}^n \sum_{j=1}^n a_i b_j \, \varphi(\vv i, \vv j) = \varphi(\v, \w), \]
da cui la tesi.
\end{solution}
\begin{proposition} Sono equivalenti le seguenti affermazioni:
\begin{enumerate}[(i)]
\item $V$ e $V'$ sono isometrici;
\item $\forall$ base $\basis$ di $V$, base $\basis'$ di $V'$,
$M_\basis(\varphi)$ e $M_{\basis'}(\varphi')$ sono congruenti;
\item $\exists$ base $\basis$ di $V$, base $\basis'$ di $V'$ tale che
$M_\basis(\varphi)$ e $M_{\basis'}(\varphi')$ sono congruenti.
\end{enumerate}
\end{proposition}
\begin{proof} Se $V$ e $V'$ sono isometrici, sia $f : V \to V'$ un'isometria. Sia $\basisC = \{ \vv 1, \ldots, \vv n \}$ una base di $V$. Allora, poiché $f$ è anche un isomorfismo, $\basisC' = f(\basisC)$ è una base di $V$ tale che
$\varphi(\vv i, \vv j) = \varphi'(f(\vv i), f(\vv j))$ $\forall 1 \leq i, j \leq n$. Pertanto $M_\basisC(\varphi) = M_{\basisC'}(\varphi')$. Si conclude allora che, cambiando base in $V$ (o in $V'$), la matrice associata
al prodotto scalare varia per congruenza dalla formula di cambiamento di base per il prodotto scalare, da cui si ricava che per ogni scelta di $\basis$ base di $V$ e di $\basis'$ base di $V'$, $M_\basis(\varphi) \cong M_{\basis'}(\varphi')$. Inoltre, se tale risultato è vero per ogni $\basis$ base di $V$ e di $\basis'$ base di $V'$, dal momento che sicuramente esistono due basi $\basis$, $\basis'$ di $V$ e $V'$, vale anche (ii) $\implies$ (iii). \\
Si dimostra ora (iii) $\implies$ (i). Per ipotesi $M_\basis(\varphi) \cong M_{\basis'}(\varphi')$, quindi
$\exists P \in \GL(n, \KK) \mid M_{\basis'}(\varphi') = P^\top M_\basis(\varphi) P$. Allora $\exists$ $\basis''$
base di $V'$ tale che $P = M_{\basis''}^{\basis'}(\Idv)$, da cui $P\inv = M_{\basis'}^{\basis''}(\varphi)$. Per la formula di cambiamento di base del prodotto
scalare, $M_{\basis''}(\varphi) = (P\inv)^\top M_{\basis'} P\inv = M_\basis(\varphi)$. Detta
$\basis'' = \{ \ww 1, \ldots, \ww n \}$, si costruisce allora l'isomorfismo $f : V \to V'$ tale
che $f(\vv i) = \ww i$ $\forall 1 \leq i \leq n$.. Dal momento che per costruzione $M_\basis(\varphi) = M_{\basis''}(\varphi')$,
$\varphi(\vv i, \vv j) = \varphi'(\ww i, \ww j) = \varphi'(f(\vv i), f(\vv j))$ $\forall 1 \leq i, j \leq n$.
Si conclude dunque che $\varphi(\v, \w) = \varphi'(f(\v), f(\w))$ $\forall \v, \w \in V$, e dunque
che $f$ è un'isometria, come desiderato dalla tesi.
\end{proof}
\begin{proposition} $(V, \varphi)$ e $(V', \varphi')$ spazi vettoriali
su $\RR$ sono
isometrici $\iff$ $\varphi$ e $\varphi'$ hanno la stessa segnatura.
\end{proposition}
\begin{proof}\nl\nl
\rightproof Per la precedente proposizione, esistono due basi $\basis$ e $\basis'$, una di $V$ e una di $V'$,
tali che $M_\basis(\varphi) \cong M_{\basis'}(\varphi)$. Allora queste due matrici condividono la stessa
segnatura, e così quindi anche $\varphi$ e $\varphi'$. \\
\leftproof Se $\varphi$ e $\varphi'$ hanno la stessa segnatura, esistono due basi $\basis = \{ \vv 1, \ldots, \vv n \}$ e $\basis' = \{ \ww 1, \ldots, \ww n \}$, una
di $V$ e una di $V'$, tali che $M = M_\basis(\varphi) = M_{\basis'}(\varphi')$ e che $M$ è una matrice di
Sylvester. Allora si costruisce $f : V \to V'$ tale che $f(\vv i) = \ww i$. Esso è un isomorfismo, e per
costruzione $\varphi(\vv i, \vv j) = \varphi'(\ww i, \ww j) = \varphi'(f(\vv i), f(\vv j))$ $\forall 1 \leq i, j \leq n$, da cui
si conclude che $\varphi(\v, \w) = \varphi'(f(\v), f(\w))$ $\forall \v$, $\w \in V$, e quindi che $V$ e
$V'$ sono isometrici.
\end{proof}
% \begin{example}
% Per $\varphi = x_1 y_1 + x_2 y_2 - x_3 y_3$. %TODO: guarda dalle slide.
% \end{example}
\begin{definition} (sottospazio isotropo)
Sia $W$ un sottospazio di $V$. Allora $W$ si dice \textbf{sottospazio isotropo} di $V$
se $\restr{\varphi}{W} = 0$.
\end{definition}
\begin{remark}\nl
\li $V^\perp$ è un sottospazio isotropo di $V$. \\
\li $\vec{v}$ è un vettore isotropo $\iff$ $W = \Span(\vec v)$ è un sottospazio isotropo di $V$. \\
\li $W \subseteq V$ è isotropo $\iff$ $W \subseteq W^\perp$.
\end{remark}
\begin{proposition}
Sia $\varphi$ non degenere. Se $W$ è un sottospazio isotropo di $V$, allora
$\dim W \leq \frac{1}{2} \dim V$.
\end{proposition}
\begin{proof}
Poiché $W$ è un sottospazio isotropo di $V$, $W \subseteq W^\perp \implies \dim W \leq \dim W^\perp$.
Allora, poiché $\varphi$ è non degenere, $\dim W + \dim W^\perp = \dim V$, $\dim W \leq \dim V - \dim W$,
da cui $\dim W \leq \frac{1}{2} \dim V$.
\end{proof}
\begin{definition}
Si definisce \textbf{indice di Witt} $W(\varphi)$ di $(V, \varphi)$
come la massima dimensione di un sottospazio isotropo.
\end{definition}
\begin{remark}\nl
\li Se $\varphi > 0$ o $\varphi < 0$, $W(\varphi) = 0$.
\end{remark}
\begin{proposition}
Sia $\KK = \RR$. Sia $\varphi$ non degenere e sia $\sigma(\varphi) = (\iota_+(\varphi), \iota_-(\varphi), 0)$. Allora
$W(\varphi) = \min\{\iota_+(\varphi), \iota_-(\varphi)\}$.
\end{proposition}
\begin{proof}
Senza perdità di generalità si assuma $\iota_-(\varphi) \leq \iota_+(\varphi)$ (il caso $\iota_-(\varphi) > \iota_+(\varphi)$ è analogo). Sia $W$ un sottospazio con $\dim W > \iota_-(\varphi)$. Sia $W^+$
un sottospazio con $\dim W^+ = \iota_+(\varphi)$ e $\restr{\varphi}{W^+} > 0$. Allora, per la formula
di Grassmann, $\dim W + \dim W^+ > n \implies \dim W + \dim W^+ > \dim W + \dim W^+ - \dim (W \cap W^+) \implies \dim (W \cap W^+) > 0$. Quindi $\exists \w \in W$, $\w \neq \vec 0$ tale che $\varphi(\w, \w) > 0$, da cui
si ricava che $W$ non è isotropo. Pertanto $W(\varphi) \leq \iota_-(\varphi)$. \\
Sia $a := \iota_+(\varphi)$ e sia $b := \iota_-(\varphi)$.
Sia ora $\basis = \{ \vv 1, \ldots, \vv a, \ww 1, \ldots, \ww b \}$ una base tale per cui $M_\basis(\varphi)$ è la matrice di Sylvester per $\varphi$. Siano $\vv 1$, ..., $\vv a$ tali che $\varphi(\vv i, \vv i) = 1$
con $1 \leq i \leq a$. Analogamente siano $\ww 1$, ..., $\ww b$ tali che $\varphi(\ww i, \ww i) = -1$ con
$1 \leq i \leq b$. Detta allora $\basis' = \{ \vv 1 ' := \vv 1 + \ww 1, \ldots, \vv b ' := \vv b + \ww b \}$, sia $W = \Span(\basis')$. \\
Si osserva che $\basis'$ è linearmente indipendente, e dunque che $\dim W = \iota_-$. Inoltre
$\varphi(\vv i ', \vv j ') = \varphi(\vv i + \ww i, \vv j + \ww j)$. Se $i \neq j$, allora
$\varphi(\vv i ', \vv j ') = 0$, dal momento che i vettori di $\basis$ sono a due a due ortogonali
tra loro. Se invece $i = j$, allora $\varphi(\vv i ', \vv j ') = \varphi(\vv i, \vv i) + \varphi(\ww i, \ww i) = 1-1=0$. Quindi $M_{\basis'}(\restr{\varphi}{W}) = 0$, da cui si conclude che $\restr{\varphi}{W} = 0$.
Pertanto $W(\varphi) \geq i_-(\varphi)$, e quindi $W(\varphi) = i_-(\varphi)$, da cui la tesi.
\end{proof}
\end{document}